Line integral convert to polar coordinates

Click For Summary
SUMMARY

The discussion focuses on calculating the work done by the force field $\vec{F}=(5x-8y\sqrt{x^2+y^2})\vec{i}+(4x+10y\sqrt{x^2+y^2})\vec{j}+z\vec{k}$ along a specified path $\vec{r}=\frac{1}{2}\cos(t)\vec{i}+\frac{1}{2}\sin(t)\vec{j}+4\arctan(t)\vec{k}$. The line integral is set up correctly, resulting in a value of approximately 5.86436. Participants question whether the integral can be simplified by converting to polar coordinates, indicating a potential area for further exploration in the solution process.

PREREQUISITES
  • Understanding of vector fields and line integrals
  • Familiarity with polar coordinates and their applications
  • Knowledge of calculus, specifically integration techniques
  • Proficiency in using trigonometric identities in integrals
NEXT STEPS
  • Learn how to convert Cartesian coordinates to polar coordinates in vector fields
  • Study the application of Green's Theorem in line integrals
  • Explore the use of parametric equations in calculating line integrals
  • Investigate advanced integration techniques for complex integrals
USEFUL FOR

Students and professionals in mathematics, physics, and engineering who are working with vector calculus and line integrals, particularly those seeking to optimize their integration techniques or understand the implications of coordinate transformations.

Scott77
Messages
7
Reaction score
0

Homework Statement


I need to find the work done by the force field:
$$\vec{F}=(5x-8y\sqrt{x^2+y^2})\vec{i}+(4x+10y\sqrt{x^2+y^2})\vec{j}+z\vec{k}$$
moving a particle from a to b along a path given by:
$$\vec{r}=\frac{1}{2}\cos(t)\vec{i}+\frac{1}{2}\sin(t)\vec{j}+4\arctan(t)\vec{k}$$

The Attempt at a Solution


So I set up my line integral:
$$\vec{F}(\vec{r}(t))=(\frac{5}{2}\cos(t)-2\sqrt{2}\sin(t))\vec{i}+(2\cos(t)+\frac{5\sqrt{2}}{2}\sin(t))\vec{j}+(4\arctan(t))\vec{k}$$
$$\vec{r'}(t)=\left(-\frac{1}{2}\sin(t)\right)\vec{i}+\left(\frac{1}{2}\cos(t)\right)\vec{j}+\left(\frac{4}{t^2+1}\right)\vec{k}$$
$$\int_0^{1}\left(-\frac{5+5\sqrt{2}}{4}\sin(t)\cos(t)+\sqrt{2}\sin^2(t)+\cos^2(t)+\frac{16\arctan(t)}{t^2+1}\right)\; \text{d}t=5.86436$$

I have left a lot of steps out, it gets messy! Could this problem be solved by reducing the integral to polar coordinates?
 
Physics news on Phys.org
Scott77 said:

Homework Statement


I need to find the work done by the force field:
$$\vec{F}=(5x-8y\sqrt{x^2+y^2})\vec{i}+(4x+10y\sqrt{x^2+y^2})\vec{j}+z\vec{k}$$
moving a particle from a to b along a path given by:
$$\vec{r}=\frac{1}{2}\cos(t)\vec{i}+\frac{1}{2}\sin(t)\vec{j}+4\arctan(t)\vec{k}$$

The Attempt at a Solution


So I set up my line integral:
$$\vec{F}(\vec{r}(t))=(\frac{5}{2}\cos(t)-2\sqrt{2}\sin(t))\vec{i}+(2\cos(t)+\frac{5\sqrt{2}}{2}\sin(t))\vec{j}+(4\arctan(t))\vec{k}$$
$$\vec{r'}(t)=\left(-\frac{1}{2}\sin(t)\right)\vec{i}+\left(\frac{1}{2}\cos(t)\right)\vec{j}+\left(\frac{4}{t^2+1}\right)\vec{k}$$
$$\int_0^{1}\left(-\frac{5+5\sqrt{2}}{4}\sin(t)\cos(t)+\sqrt{2}\sin^2(t)+\cos^2(t)+\frac{16\arctan(t)}{t^2+1}\right)\; \text{d}t=5.86436$$

I have left a lot of steps out, it gets messy! Could this problem be solved by reducing the integral to polar coordinates?
What makes you think you didn't use polar coordinates to get your result?
 
Question: A clock's minute hand has length 4 and its hour hand has length 3. What is the distance between the tips at the moment when it is increasing most rapidly?(Putnam Exam Question) Answer: Making assumption that both the hands moves at constant angular velocities, the answer is ## \sqrt{7} .## But don't you think this assumption is somewhat doubtful and wrong?

Similar threads

  • · Replies 4 ·
Replies
4
Views
1K
  • · Replies 9 ·
Replies
9
Views
1K
Replies
3
Views
2K
Replies
20
Views
2K
  • · Replies 9 ·
Replies
9
Views
2K
  • · Replies 14 ·
Replies
14
Views
2K
Replies
1
Views
1K
  • · Replies 5 ·
Replies
5
Views
2K
  • · Replies 9 ·
Replies
9
Views
2K
  • · Replies 1 ·
Replies
1
Views
2K